Last visit was: 25 Apr 2024, 21:15 It is currently 25 Apr 2024, 21:15

Close
GMAT Club Daily Prep
Thank you for using the timer - this advanced tool can estimate your performance and suggest more practice questions. We have subscribed you to Daily Prep Questions via email.

Customized
for You

we will pick new questions that match your level based on your Timer History

Track
Your Progress

every week, we’ll send you an estimated GMAT score based on your performance

Practice
Pays

we will pick new questions that match your level based on your Timer History
Not interested in getting valuable practice questions and articles delivered to your email? No problem, unsubscribe here.
Close
Request Expert Reply
Confirm Cancel
SORT BY:
Date
Tags:
Difficulty: 505-555 Levelx   Geometryx               
Show Tags
Hide Tags
Math Expert
Joined: 02 Sep 2009
Posts: 92915
Own Kudos [?]: 619048 [36]
Given Kudos: 81595
Send PM
Most Helpful Reply
Manager
Manager
Joined: 14 Apr 2017
Posts: 79
Own Kudos [?]: 863 [12]
Given Kudos: 565
Location: Hungary
GMAT 1: 760 Q50 V42
WE:Education (Education)
Send PM
General Discussion
GMAT Club Legend
GMAT Club Legend
Joined: 18 Aug 2017
Status:You learn more from failure than from success.
Posts: 8019
Own Kudos [?]: 4098 [1]
Given Kudos: 242
Location: India
Concentration: Sustainability, Marketing
GMAT Focus 1:
545 Q79 V79 DI73
GPA: 4
WE:Marketing (Energy and Utilities)
Send PM
Manager
Manager
Joined: 25 Jul 2018
Posts: 53
Own Kudos [?]: 401 [2]
Given Kudos: 257
Location: Uzbekistan
Concentration: Finance, Organizational Behavior
GRE 1: Q168 V167
GPA: 3.85
WE:Project Management (Investment Banking)
Send PM
Re: In the figure above, ΔPQR has angle measures as shown. Is x < y ? [#permalink]
2
Kudos
Bunuel wrote:

In the figure above, ΔPQR has angle measures as shown. Is x < y ?

(1) PQ = QR
(2) PR > QR


DS27502.01
OG2020 NEW QUESTION


Attachment:
2019-04-26_1253.png



Is x < y?

1. PQ = QR

The greater the angle is the greater is the side opposite to it. If the triangle has two equal sides, it is an isosceles triangle with two equal angles opposite to those sides. Since PQ = QR, x = 58. Thus y = 180 - 58 - 58 = 64. x < y. Sufficient


2. PR > QR

Since the side opposite to y is greater than the side opposite to x, y must be greater than x. Accordingly, x < y. Sufficient
GMAT Tutor
Joined: 15 Aug 2017
Posts: 78
Own Kudos [?]: 597 [3]
Given Kudos: 75
GMAT 1: 780 Q49 V51
WE:Education (Education)
Send PM
In the figure above, ΔPQR has angle measures as shown. Is x < y ? [#permalink]
3
Kudos
Expert Reply
Quote:
In the figure above, ΔPQR has angle measures as shown. Is x < y ?

(1) PQ = QR
(2) PR > QR


In this case, Statement (1) tells us that triangle PQR is an isosceles triangle, with sides PQ=QR, thus corresponding angles PRQ and QPR are also equal. So, x must also be 58 degrees, and since the sum of the angles of a triangle must be 180 degrees, angle y must be 180-58-58, or 64 degrees, answering the question yes. (Sufficient) Keep in mind, on test day, as soon as we know that statement one will give us precisely one situation for the angle measures, we don't actually need to make the calculation, regardless of whether the situation answers the question "yes" or "no," if we have one distinct possibility for the angle measures, the statement will be sufficient to answer our question.

Statement (2) tells us that the length of PR is greater than the length of QR, thus, the corresponding opposite angle from PR, angle PQR, must be greater than the angle opposite QR, angle QPR - so, yes, y is once again greater than x. (Sufficient)

This question really just comes down to understanding the relationships between side lengths and their corresponding angle measures, and utilizing a bit of data sufficiency strategy to recognize that when we are given enough information to solve for precisely one situation, we'll be able to answer the question, often without having to actually make the calculation! :)
Target Test Prep Representative
Joined: 14 Oct 2015
Status:Founder & CEO
Affiliations: Target Test Prep
Posts: 18761
Own Kudos [?]: 22055 [0]
Given Kudos: 283
Location: United States (CA)
Send PM
Re: In the figure above, ΔPQR has angle measures as shown. Is x < y ? [#permalink]
Expert Reply
Bunuel wrote:

In the figure above, ΔPQR has angle measures as shown. Is x < y ?

(1) PQ = QR
(2) PR > QR


DS27502.01
OG2020 NEW QUESTION


Attachment:
2019-04-26_1253.png



We are given that angle QRP = 58 and need to determine whether x is less than y.

Statement One Alone:

PQ = QR

Since PQ = QR, x = 58.

Thus, y = 180 - 58 - 58 = 64.

So we see that x is less than y.

Statement one alone is sufficient to answer the question.

Statement Two Alone:

PR > QR

Since PR is greater than QR, the angle opposite PR is greater than the angle opposite QR.

Thus, y > x, so we see that statement two alone is also sufficient to answer the question.

Answer: D
GMAT Club Legend
GMAT Club Legend
Joined: 19 Dec 2014
Status:GMAT Assassin/Co-Founder
Affiliations: EMPOWERgmat
Posts: 21846
Own Kudos [?]: 11666 [1]
Given Kudos: 450
Location: United States (CA)
GMAT 1: 800 Q51 V49
GRE 1: Q170 V170
Send PM
Re: In the figure above, ΔPQR has angle measures as shown. Is x < y ? [#permalink]
1
Kudos
Expert Reply
Hi All,

We're told that the triangle PQR has angle measures as shown. We're asked if X is less than Y. This is a YES/NO question and is based around a specific Triangle rule (re: in simple terms, the bigger the angle, the bigger the side that's across from it). Since Angle R is 58 degrees, the sum of Angles X and Y is 122 degrees. If X and Y are EQUAL, then the sides across from them will also be EQUAL.

(1) PQ = QR

With the information in Fact 1, we know that two of the sides are EQUAL, so the angles across from them will also be EQUAL. Thus, Angle X = 58 degrees. By extension, Angle Y = 122 - 58 = 64 degrees and the answer to the question is YES.
Fact 1 is SUFFICIENT

(2) PR > QR

Fact 2 tells us that the side across from Angle Y is GREATER than the side across from Angle X, so Y is greater than X and the answer to the question is YES.
Fact 2 is SUFFICIENT

Final Answer:

GMAT assassins aren't born, they're made,
Rich
e-GMAT Representative
Joined: 04 Jan 2015
Posts: 3726
Own Kudos [?]: 16841 [0]
Given Kudos: 165
Send PM
In the figure above, ΔPQR has angle measures as shown. Is x < y ? [#permalink]
Expert Reply

Solution



Steps 1 & 2: Understand Question and Draw Inferences

In this question, we are given
    • The figure of a triangle PQR, where angle P = x°, angle Q = y° and angle R = 58°

From this information, we need to determine
    • If x < y or not.

As PQR is a triangle, the sum of the three angles must be equal to 180°
    • x° + y° + 58° = 180°
Or, x° + y° = 180° - 58° = 122°

Hence, to determine whether x < y, we need to know the value of either x or y.
With this information, let us now analyse the individual statements.

Step 3: Analyse Statement 1

As per the information given in statement 1, PQ = QR.
    • Therefore, we can say angle QPR = angle QRP
Or, x° = 58°
As we can determine the value of x, we can also determine the value of y and find out whether x < y or not.
Hence, statement 1 is sufficient to answer the question.

Step 4: Analyse Statement 2

As per the information given in statement 2, PR > QR.
    • Therefore, the angle opposite to PR > the angle opposite to QR
Or, y > x
As we can determine whether x < y or not, statement 2 is sufficient to answer the question.

Step 5: Combine Both Statements Together (If Needed)

Since we could determine the answer from either of the statements individually, this step is not required.

Hence, the correct answer is option D.

avatar
Intern
Intern
Joined: 15 May 2019
Posts: 4
Own Kudos [?]: 1 [0]
Given Kudos: 2
Send PM
Re: In the figure above, ΔPQR has angle measures as shown. Is x < y ? [#permalink]
Foe me definitely D.
Intern
Intern
Joined: 25 Jan 2016
Posts: 48
Own Kudos [?]: 28 [0]
Given Kudos: 65
Location: India
Schools: Alberta '23
GPA: 3.9
WE:Engineering (Energy and Utilities)
Send PM
Re: In the figure above, ΔPQR has angle measures as shown. Is x < y ? [#permalink]
OG 21 explanation

Given that PQ = QR, it follows that the angle at vertex R has the same measure as the angle at vertex P, or 58 = x. Therefore, using x + y + 58 = 180 gives 58 + y + 58 = 180, or y = 64. Since the values of both x and y are now known, it can be determined whether x < y; SUFFICIENT.
Given that PR > QR, it follows that the measure of the angle at vertex Q is greater than the measure of the angle at vertex P, or y > x; SUFFICIENT.
Director
Director
Joined: 14 Jul 2010
Status:No dream is too large, no dreamer is too small
Posts: 972
Own Kudos [?]: 4928 [0]
Given Kudos: 690
Concentration: Accounting
Send PM
Re: In the figure above, ΔPQR has angle measures as shown. Is x < y ? [#permalink]
Top Contributor
Bunuel wrote:

In the figure above, ΔPQR has angle measures as shown. Is x < y ?

(1) PQ = QR
(2) PR > QR
Attachment:
2019-04-26_1253.png



(1) so x 58°, y will greater than x; Sufficient.

(2) A side represents it's opposite angle. so y>x. Sufficient.

The answer is D
Tutor
Joined: 17 Jul 2019
Posts: 1304
Own Kudos [?]: 2287 [1]
Given Kudos: 66
Location: Canada
GMAT 1: 780 Q51 V45
GMAT 2: 780 Q50 V47
GMAT 3: 770 Q50 V45
Send PM
Re: In the figure above, ΔPQR has angle measures as shown. Is x < y ? [#permalink]
1
Kudos
Expert Reply
Video solution from Quant Reasoning:
Subscribe for more: https://www.youtube.com/QuantReasoning? ... irmation=1
GMAT Club Legend
GMAT Club Legend
Joined: 08 Jul 2010
Status:GMAT/GRE Tutor l Admission Consultant l On-Demand Course creator
Posts: 5960
Own Kudos [?]: 13387 [1]
Given Kudos: 124
Location: India
GMAT: QUANT+DI EXPERT
Schools: IIM (A) ISB '24
GMAT 1: 750 Q51 V41
WE:Education (Education)
Send PM
Re: In the figure above, ΔPQR has angle measures as shown. Is x < y ? [#permalink]
1
Kudos
Expert Reply
Bunuel wrote:

In the figure above, ΔPQR has angle measures as shown. Is x < y ?

(1) PQ = QR
(2) PR > QR


DS27502.01
OG2020 NEW QUESTION


Attachment:
2019-04-26_1253.png


Wanna make solving the Official Questions interesting???


Click here and solve 1000+ Official Questions with Video solutions as Timed Sectional Tests
and Dedicated Data Sufficiency (DS) Course


Answer: Option D

Video solution by GMATinsight



Get TOPICWISE: Concept Videos | Practice Qns 100+ | Official Qns 50+ | 100% Video solution CLICK HERE.
Two MUST join YouTube channels : GMATinsight (1000+ FREE Videos) and GMATclub :)
User avatar
Non-Human User
Joined: 09 Sep 2013
Posts: 32681
Own Kudos [?]: 822 [0]
Given Kudos: 0
Send PM
Re: In the figure above, PQR has angle measures as shown. Is x < y ? [#permalink]
Hello from the GMAT Club BumpBot!

Thanks to another GMAT Club member, I have just discovered this valuable topic, yet it had no discussion for over a year. I am now bumping it up - doing my job. I think you may find it valuable (esp those replies with Kudos).

Want to see all other topics I dig out? Follow me (click follow button on profile). You will receive a summary of all topics I bump in your profile area as well as via email.
GMAT Club Bot
Re: In the figure above, PQR has angle measures as shown. Is x < y ? [#permalink]
Moderator:
Math Expert
92915 posts

Powered by phpBB © phpBB Group | Emoji artwork provided by EmojiOne